Difference between revisions of "1989 AIME Problems/Problem 11"

m
Line 4: Line 4:
 
== Solution ==
 
== Solution ==
 
{{solution}}
 
{{solution}}
 
+
It is obvious that there will be n+1 values equal to one and n values each of 1000, 999, 998... .  It is fairly easy to find the maximum.  Try n=1 (yields 924) n=2 (yields 942) n=3 (yields 947) and n=4 (yields 944).  The maximum difference occered at n=3, so the answer is 947.
 
== See also ==
 
== See also ==
 
* [[1989 AIME Problems/Problem 12|Next Problem]]
 
* [[1989 AIME Problems/Problem 12|Next Problem]]
 
* [[1989 AIME Problems/Problem 10|Previous Problem]]
 
* [[1989 AIME Problems/Problem 10|Previous Problem]]
 
* [[1989 AIME Problems]]
 
* [[1989 AIME Problems]]

Revision as of 20:34, 6 March 2007

Problem

A sample of 121 integers is given, each between 1 and 1000 inclusive, with repetitions allowed. The sample has a unique mode (most frequent value). Let $D^{}_{}$ be the difference between the mode and the arithmetic mean of the sample. What is the largest possible value of $\lfloor D^{}_{}\rfloor$? (For real $x^{}_{}$, $\lfloor x^{}_{}\rfloor$ is the greatest integer less than or equal to $x^{}_{}$.)

Solution

This problem needs a solution. If you have a solution for it, please help us out by adding it. It is obvious that there will be n+1 values equal to one and n values each of 1000, 999, 998... . It is fairly easy to find the maximum. Try n=1 (yields 924) n=2 (yields 942) n=3 (yields 947) and n=4 (yields 944). The maximum difference occered at n=3, so the answer is 947.

See also